Wirkungsprinzip für den Elektromagnetismus und die Schwerkraft

Hier ist die Formel für den Spannungsenergietensor:

T μ v = 2 | det G | δ S E M δ G μ v
(Dies folgt aus der Variation der Gesamtwirkung S = S H + S E M , Wo S H = C 4 16 π G R | det G μ v | D 4 X ist die Hilbert-Aktion und ergibt die Einstein-Gleichungen, und S E M sind andere Terme im Lagrange, die zur rechten Seite der Einstein-Gleichungen in Form von beitragen T μ v oben.) Der Lagrange-Operator des elektromagnetischen Feldes ist:

S E M 1 = 1 4 μ 0 F a β F a β | det G | D 4 X

und unter Verwendung der obigen Formel erhalten wir für den Spannungsenergietensor:

T μ v = 1 μ 0 ( F μ β F v β 1 4 F a β F a β G μ v )
das ist das richtige elmag. Spannungsenergietensor. Der Interaktionsteil des elmag. Lagrange ist
S E M 2 = J μ A μ | det G | D 4 X
und wenn wir dies als Funktion von interpretieren G μ v es würde auch wie folgt zum Spannungsenergietensor beitragen:
δ S E M 2 = δ J μ A μ | det G | D 4 X = δ ( G μ v J μ A v | det G | ) D 4 X =
= ( δ G μ v ) ( J μ | det G | ) ) A v D 4 X

bei dem die J μ | det G | wird als Stromdichte behandelt (und ist somit nicht abhängig von G μ v bei Variation), wäre aber eindeutig der dazu korrespondierende Spannungsenergietensor:

T μ v = 2 J μ A v
(Möglicherweise trägt nur der symmetrische Anteil bei, weil sich der antisymmetrische Anteil mit aufhebt G μ v , so würden wir bekommen T μ v = J μ A v + J v A μ .) In jedem Fall sollten solche Terme dann auf der rechten Seite der Einstein-Gleichungen erscheinen. Ich glaube jedoch nicht, dass dies richtig ist.

Weiß jemand, was hier falsch ist?

Die Variablen, die Sie variieren, sind A μ Und G μ v . Seit A μ Natürlich erscheint mit einem abgesenkten Index die Variation von J μ A μ in Bezug auf die inverse Metrik ist Null, es sei denn, es gibt Faktoren von G μ v versteckt in der J μ . Außerdem fehlt dir der Faktor R das gibt Ihnen die Dynamik der Schwerkraft, obwohl das beabsichtigt sein könnte.
Hallo Jerry, ich denke, das ist es. (Ja, ich habe den Faktor übersprungen R für Einsteins Gleichungen hier.) Aber woher weiß ich, dass ich manchmal die verwenden muss G μ v Faktor (und dann variiere), und manchmal nicht? Es scheint mir, dass es völlig willkürlich ist.
Sie müssen eine Entscheidung darüber treffen, welchen Begriff Sie variieren werden. Es gibt keinen Faktor G μ v In A μ J μ , Weil A μ erscheint natürlich mit einem Down-Index. Dieser Term ist eine Form, die auf einen Vektor wirkt, was unabhängig von der Metrik geschieht. Und J μ ist natürlich ein Vektor und keine Einsform, weil es typischerweise definiert ist durch J μ δ L C H A R G e D M A T T e R δ A μ .
Daher gibt es im natürlichen Sinne in beiden Termen keine Abhängigkeit von der Metrik.
Jerry, vielen Dank. Was du schreibst macht für mich sehr viel Sinn. Aber was ist mit einem Begriff wie P μ P μ (siehe physical.stackexchange.com/questions/17604/… )? Könnte es daran liegen, dass es sich um einen gemischten Begriff handelt P μ Und P μ man muss es umwandeln, um es nur zu verwenden P μ (sagen wir) und man bekommt G μ v P μ P v und die Metrik ist (natürlich?) da. Während für einen Begriff wie J μ A μ = J μ A μ , die Metrik ist natürlich nicht da. Aber ich denke, ich muss das wirklich vorher annehmen (angeben), oder?
Noch eine Frage: warum A μ erscheinen natürlich mit dem niedrigeren Index? Ist es so F μ v = μ A v v A μ hat beide Down-Indizes? (Das Derivat ist natürlich mit einem Abwärtsindex, und man muss die Metrik verwenden, um ihn zu erhöhen, das ist mir klar.)
@Ondrej: Sie erhalten die gleiche Spannungstensorquelle für Einstein-Gleichungen mit beiden Indexpositionen, wie ich in meiner Antwort beschreibe. Der untere Index ergibt eine etwas einfachere Variante, das ist alles.
@Ondrej: Aber Sie müssen sich entscheiden, die eine oder andere Wahl zu variieren. Sobald Sie das behoben haben, haben Sie entschieden, welche Freiheitsgrade Feld sind und welche aus Inversionsoperatoren und Erhöhungsoperationen usw. bestehen.
Vielleicht finden Sie Abschnitt 48 von „Die Theorie von Raum, Zeit und Gravitation“ von VA Fock nützlich.

Antworten (3)

Die Kommentare von Jerry Schirmer sagen Ihnen die Hauptidee, aber ich möchte sie expliziter und in Antwortform geben. Beim Variieren der Aktion bzgl A μ , gibt die metrische Variation den von Ihnen erwähnten unerwünschten Begriff nicht an. Aber wenn Sie in Bezug auf variieren A μ , erhalten Sie diesen Term, und er sollte nicht auf der rechten Seite von Einsteins Gleichungen erscheinen, da wir diese Gleichungen bereits aus der A 1-Form-Version der Variation kennen.

Aber die Bewegungsgleichungen sollten sich nicht darum kümmern, ob Sie in Bezug auf variieren A μ oder bzgl A μ , sollten Sie die gleichen Gleichungen erhalten. Formal

δ S = δ S δ G μ v δ G μ v + δ S δ A μ δ A μ

Und die Einstein-Gleichungen sind die Koeffizienten von δ G , während die (vektorpotentialfreien) Maxwell-Gleichungen die Koeffizienten von sind δ A .

Die Variationen in A μ , G μ v lässt sich leicht durch Variationen in ausdrücken A μ , G μ v ,

δ A v = δ A μ G μ v + A μ δ G μ v

Was, wenn es die Gesamtvariation der Aktion ausdrückt, die Einstein- und Maxwell-Teile linear verwechselt:

δ S = ( δ S δ G μ v + δ S δ A μ A v ) δ G μ v + δ S δ A μ G μ v δ A v

Wobei die Variationsableitungen alle alten Variationsableitungen in Bezug auf das Paar sind G μ v , A μ hält den anderen fest. Diese linearen Kombinationen ergeben die neuen Variationen. Es ist trivial zu sehen, dass die neuen Bewegungsgleichungen genau dann erfüllt sind, wenn die alten es sind, also hat sich nichts geändert.

Die neuen Maxwell-Gleichungen sind nach Multiplikation mit der inversen Metrik die gleichen wie die alten. Aber die neue Einstein-Gleichung enthält einen zusätzlichen Quellterm:

δ S δ A μ A v

Dieser zusätzliche Quellterm ist nach den Maxwell-Gleichungen aber offensichtlich null δ S δ A μ beinhaltet den Begriff J μ , daher erscheint hier der Begriff, der Sie störte. Diese Variation ergibt eine rechte Seite von Einsteins Gleichungen, die eine zusätzliche Spannung enthält, die den Quellterm in Form der Maxwell-Gleichung multipliziert mit dem Vektorpotential enthält

( D μ F μ v J v ) A μ

Aber jetzt ist es offensichtlich, dass der Stressbeitrag verschwindet (wie immer, weil dies nur eine Variation in Bezug auf verschiedene Variablen derselben Aktion ist).

Über die Dichtevariationen

Jons Antwort berechnet einen zusätzlichen Term aus dem Variieren G , aber dieser Begriff ist nicht vorhanden. Dies ist aus dem in der Antwort auf diese Frage erläuterten Grund: Lagrangeian for Relativistic Dust Derivation Questions .

Wenn Sie die Metrik mit EM und beispielsweise einer geladenen Staubquelle variieren, halten Sie J G Fest. Dies ist aus dem gleichen Grund, aus dem die Impulsdichte konstant gehalten wird, Sie halten die Anzahl der Weltlinien konstant, wenn Sie g variieren, sodass die konservierten Ströme und Ladungen bei metrischen Variationen erhalten bleiben.

Danke! Ich habe über Ihre Antwort in "Lagrangeian for Relativistic Dust Derivation Questions" nachgedacht und mir ist jetzt klar, dass Sie halten müssen J G Fest. Sie erhalten also keine zusätzlichen Bedingungen, die durch das Variieren entstehen G , das ist jetzt klar. Ich werde einige Zeit über den Rest Ihrer Antwort nachdenken (und es selbst ausarbeiten, um es gut zu verstehen), bevor ich es akzeptiere.
Korrektur: δ A N u oben sollte sein δ ( A μ G μ v ) .
@Ondrej: ja, das war ein Textfehler für δ A v Fest.

Ich denke, das im Ausdruck

S E M 2 = J μ A μ G D 4 X
Sie müssen auch die Determinante herleiten. Wenn Sie dies tun, erhalten Sie
1 2 G ( J v A μ + J μ A v ) δ G μ v 1 2 G μ v G J A δ G μ v .
Sie werden also bekommen
T μ v ( 2 ) = 2 G δ S E M 2 δ G μ v = ( J v A μ + J μ A v ) + G μ v J A .
Dies sollte das richtige Ergebnis sein.

Die Stress-Energie-Formel ist T μ v ( 2 ) = 2 G δ S E M 2 δ G μ v also bekommst du das nicht 1 2 in der endgültigen Antwort.
Wenn die Determinante ebenfalls variiert werden muss, ist Ihre Ableitung korrekt (obwohl Dirac sie nicht variiert). Allerdings habe ich noch nie zuvor einen solchen Stress-Energie-Tensor gesehen (zusammengesetzt aus J Und A ). Nur T μ v = 1 μ 0 ( F μ β F v β 1 4 F a β F a β G μ v ) . Hast du einen Literaturhinweis dazu? Was ist seine physikalische Interpretation?
Kleiner Kommentar: Ich denke, Sie haben das Gesamtzeichen falsch verstanden, es gibt ein Minus in der Aktion, ein Minus in der Definition des Stressenergietensors und ein Minus durch Variieren der Determinante, also denke ich, dass die richtige Antwort ist
T μ v ( 2 ) = 2 G δ S E M 2 δ G μ v = + ( J v A μ + J μ A v ) G μ v J A .
. Aber egal, das Vorzeichen ist für diese Diskussion nicht wichtig.
Danke, ich habe den Faktor 1/2 korrigiert. Sie können hier en.wikipedia.org/wiki/Maxwell%27s_equations_in_curved_spacetime überprüfen . Der Wechselwirkungsterm liefert einen Beitrag zur rechten Seite der Einstein-Gleichungen. Dies kann so verstanden werden, dass, wenn Ladungsmaterie vorhanden ist, dies einen Beitrag zum Spannungsenergietensor hinzufügt.
Danke. Nur damit wir uns verstehen, sagen Sie, dass die rechte Seite der Einstein-Gleichungen Folgendes enthalten wird: T μ v ( 1 ) + T μ v ( 2 ) , Wo T μ v ( 1 ) = 1 μ 0 ( F μ β F v β 1 4 F a β F a β G μ v ) Und T μ v ( 2 ) = ( J v A μ + J μ A v ) + G μ v J A . Ich ging zum Wiki, und ich kann dort die sehen T μ v ( 1 ) Begriff, aber nicht die T μ v ( 2 ) Begriff. Übersehe ich etwas?
Das meine ich. Materie kann auch geladen sein und dies muss in Einstein-Gleichungen berücksichtigt werden.
Danke für die Klarstellung. Ich habe Zweifel bzgl T μ v ( 2 ) term -- Ich habe es noch nie gesehen (lass es mich wissen, wenn ich es im Wiki verpasst habe). Es spielt eigentlich keine Rolle, ob Sie die Determinante variieren oder nicht (es addiert einfach die J A Begriff). Geladene Materie beeinflusst das Elmag. Feld selbst ( A μ bzw. F μ v ), die Teil der ist T μ v ( 1 ) Tensor, Ladung Angelegenheit wird durch die Rechnung F μ v Feld schon. Aber es ist für mich verwirrend, warum es einen solchen Begriff nicht gibt, weil es scheint, dass das Variationsprinzip es nahe legt.
Deine letzten Sätze habe ich nicht ganz verstanden. Wenn Sie geladene Materie haben, zum Beispiel ein kugelförmiges geladenes Objekt, müssen Sie dies in der Einstein-Gleichung berücksichtigen und es ist nicht im em-Spannungsenergietensor enthalten. Sie können dies besser verstehen, wenn Sie bedenken, dass die kovariante Ableitung der rechten Seite der Einstein-Gleichungen Ihnen die Bewegungsgleichungen für die Materie liefert. Sie müssen sich durchsetzen μ T μ v = 0 .
Jon, so wie ich es verstehe, erzeugt geladene Materie das Elmag-Feld F μ v durch die rechte Seite der Maxwell-Gleichungen (das enthält J μ ). Als solches wird Ladungsmaterie erzeugt F μ v . Das F μ v gehört schon dazu T μ v ( 1 ) , und als solches ist es bereits in den Einstein-Gleichungen berücksichtigt. Mit anderen Worten, die geladene Materie krümmt die Raumzeit aufgrund ihrer Masse und auch aufgrund ihrer Ladung/Ströme. Ob es auch den Begriff gibt T μ v ( 2 ) ist ein anderes Thema. Wenn Sie Literatur oder Webseiten darüber kennen, weisen Sie mich bitte darauf hin.

Eigentlich ist Ihre ursprüngliche Formel falsch: die δ ( S E M ) / δ ( G ( u , v ) ) sollte sein δ ( L E M / δ ( G ( u , v ) ) , Wo L (der Lagrange) ist das Ding, das über den Raum integriert wird, um die Aktion zu erhalten S .

Es ist eine Apfel-Orangen-Sache: Die Aktion ist das Integral der Lagrange-Funktion, δ (Aktion) bleibt eine Zahl, nicht wie gefordert eine Leerzeichenfunktion.

Interessanterweise haben sowohl Walds Buch als auch MTW-Buch diesen Fehler, genau wie Sie, also ist es verständlich. Wenn Sie sich andere Quellen zur Lagrange-Feldtheorie oder sogar den Wikipedia-Artikel zum Stress-Energie-Tensor unter dem Hilbert-Spannungs-Energie-Tensor ansehen, haben sie es richtig, indem sie das Delta der Dichtefunktion L anstelle des Integrals davon nehmen, S. bei der Definition des Spannungs-Energie-Tensors (der eine Raumfunktion ist).

Dies hat keine Auswirkungen auf Ihre Bearbeitung des Problems, aber es ist bemerkenswert, dass diese beiden weit verbreiteten Lehrbücher über GR diese Sache schreiben würden, die keinen Sinn macht, und jeder akzeptiert es einfach und geht so vor, als ob es Sinn hätte. Studenten sind sehr tolerante Menschen.

Können Sie genauer werden und die Latex-Notation verwenden, um zu schreiben, was Ihrer Meinung nach die richtige Formel sein sollte? Ich denke, wie ich es geschrieben habe, ist es richtig. Sie nehmen die Lagrange- Dichte und integrieren über die Raumzeit (4D-Integral), um die Aktion zu erhalten. Dann wenden Sie die funktionale Ableitung auf die Aktion an , nicht die Lagrange-Dichte. Soweit ich weiß, ist das zumindest die Standarddefinition.